Difference between revisions of "1986 AJHSME Problems/Problem 4"

m (Solution)
(Solution)
 
(4 intermediate revisions by 3 users not shown)
Line 7: Line 7:
 
==Solution==
 
==Solution==
  
The easiest way to do this problem is to estimate, since it says "closest to", and the multiple choices are pretty spread out.
+
<center>
 +
<math>(1.8)(40.37)\approx (1.8)(40)=72.</math>
 +
</center>
  
<math>1.8</math> is about <math>2</math>, <math>40.3</math> is about <math>40</math>, and <math>.07</math> is about <math>0</math>.
+
Approximating <math>40.37</math> instead of <math>1.8</math> is more effective because larger numbers are less affected by absolute changes (e.g <math>1001</math> is much closer relatively to <math>1000</math> than <math>2</math> is to <math>1</math>).
Putting this in, we get <math>(2)(40 + 0) = 80</math>
+
<math>74</math> is the closest to <math>72</math>, so the answer is <math>\boxed{\text{C}}</math>.
 
 
<math>80</math> is about <math>84</math>
 
 
 
<math>\boxed{\text{D}}</math>
 
  
 
==See Also==
 
==See Also==
  
[[1986 AJHSME Problems]]
+
{{AJHSME box|year=1986|num-b=3|num-a=5}}
 +
[[Category:Introductory Algebra Problems]]
 +
{{MAA Notice}}

Latest revision as of 02:17, 24 November 2017

Problem

The product $(1.8)(40.3+.07)$ is closest to

$\text{(A)}\ 7 \qquad \text{(B)}\ 42 \qquad \text{(C)}\ 74 \qquad \text{(D)}\ 84 \qquad \text{(E)}\ 737$

Solution

$(1.8)(40.37)\approx (1.8)(40)=72.$

Approximating $40.37$ instead of $1.8$ is more effective because larger numbers are less affected by absolute changes (e.g $1001$ is much closer relatively to $1000$ than $2$ is to $1$). $74$ is the closest to $72$, so the answer is $\boxed{\text{C}}$.

See Also

1986 AJHSME (ProblemsAnswer KeyResources)
Preceded by
Problem 3
Followed by
Problem 5
1 2 3 4 5 6 7 8 9 10 11 12 13 14 15 16 17 18 19 20 21 22 23 24 25
All AJHSME/AMC 8 Problems and Solutions

The problems on this page are copyrighted by the Mathematical Association of America's American Mathematics Competitions. AMC logo.png